What does $1+frac{1}{8}+frac{1}{27}+frac{1}{64}+frac{1}{125}+(frac{1}{n})^3$ equal to?












1












$begingroup$


I'm curious of what does this sum:



$1+frac{1}{8}+frac{1}{27}+frac{1}{64}+frac{1}{125}+frac{1}{216}+...+(frac{1}{n})^3$



or the Riemann zeta function: $zeta({3})$ approach. I watched a few 3Blue1Brown videos on YouTube, but it doesn't have any videos about what does that sum above approach. I don't know how to prove it, and I tried, but suddenly all of my tries lead to no results. Is this problem unsolved? If not, how would you prove it?










share|cite|improve this question











$endgroup$












  • $begingroup$
    It approaches, by definition, $zeta(3)ldots$ There's nothing to prove, except perhaps that the sum converges (obvious by integral test)
    $endgroup$
    – Brevan Ellefsen
    Nov 4 '18 at 22:30


















1












$begingroup$


I'm curious of what does this sum:



$1+frac{1}{8}+frac{1}{27}+frac{1}{64}+frac{1}{125}+frac{1}{216}+...+(frac{1}{n})^3$



or the Riemann zeta function: $zeta({3})$ approach. I watched a few 3Blue1Brown videos on YouTube, but it doesn't have any videos about what does that sum above approach. I don't know how to prove it, and I tried, but suddenly all of my tries lead to no results. Is this problem unsolved? If not, how would you prove it?










share|cite|improve this question











$endgroup$












  • $begingroup$
    It approaches, by definition, $zeta(3)ldots$ There's nothing to prove, except perhaps that the sum converges (obvious by integral test)
    $endgroup$
    – Brevan Ellefsen
    Nov 4 '18 at 22:30
















1












1








1





$begingroup$


I'm curious of what does this sum:



$1+frac{1}{8}+frac{1}{27}+frac{1}{64}+frac{1}{125}+frac{1}{216}+...+(frac{1}{n})^3$



or the Riemann zeta function: $zeta({3})$ approach. I watched a few 3Blue1Brown videos on YouTube, but it doesn't have any videos about what does that sum above approach. I don't know how to prove it, and I tried, but suddenly all of my tries lead to no results. Is this problem unsolved? If not, how would you prove it?










share|cite|improve this question











$endgroup$




I'm curious of what does this sum:



$1+frac{1}{8}+frac{1}{27}+frac{1}{64}+frac{1}{125}+frac{1}{216}+...+(frac{1}{n})^3$



or the Riemann zeta function: $zeta({3})$ approach. I watched a few 3Blue1Brown videos on YouTube, but it doesn't have any videos about what does that sum above approach. I don't know how to prove it, and I tried, but suddenly all of my tries lead to no results. Is this problem unsolved? If not, how would you prove it?







riemann-zeta






share|cite|improve this question















share|cite|improve this question













share|cite|improve this question




share|cite|improve this question








edited Jan 27 at 23:15







I-85a

















asked Nov 4 '18 at 22:27









I-85aI-85a

83




83












  • $begingroup$
    It approaches, by definition, $zeta(3)ldots$ There's nothing to prove, except perhaps that the sum converges (obvious by integral test)
    $endgroup$
    – Brevan Ellefsen
    Nov 4 '18 at 22:30




















  • $begingroup$
    It approaches, by definition, $zeta(3)ldots$ There's nothing to prove, except perhaps that the sum converges (obvious by integral test)
    $endgroup$
    – Brevan Ellefsen
    Nov 4 '18 at 22:30


















$begingroup$
It approaches, by definition, $zeta(3)ldots$ There's nothing to prove, except perhaps that the sum converges (obvious by integral test)
$endgroup$
– Brevan Ellefsen
Nov 4 '18 at 22:30






$begingroup$
It approaches, by definition, $zeta(3)ldots$ There's nothing to prove, except perhaps that the sum converges (obvious by integral test)
$endgroup$
– Brevan Ellefsen
Nov 4 '18 at 22:30












2 Answers
2






active

oldest

votes


















2












$begingroup$

$zeta(3)$ is a constant known as Apery's constant. Its value is approximately $1.2021ldots$ but as far as I know an exact value isn't known.



Some further reading:



https://en.wikipedia.org/wiki/Ap%C3%A9ry%27s_constant
http://mathworld.wolfram.com/AperysConstant.html






share|cite|improve this answer











$endgroup$













  • $begingroup$
    I don't know what it is you think "isn't quite known". The "exact value" is $zeta(3)$. Apéry proved it is irrational.
    $endgroup$
    – Robert Israel
    Nov 4 '18 at 22:49










  • $begingroup$
    "The exact value of $zeta(3)$ is $zeta(3)$" isn't exactly an enlightening answer. I mean something in terms of - at least more known - constants, i.e. how we know $zeta(2) = pi^2/6$. A "closed form" value, perhaps? Whatever the term is.
    $endgroup$
    – Eevee Trainer
    Nov 4 '18 at 22:51










  • $begingroup$
    It is not likely that $zeta(3)$ has a closed form in terms of "better-known" constants, though of course this has not been proven. We don't even have a proof that it is transcendental.
    $endgroup$
    – Robert Israel
    Nov 4 '18 at 23:00










  • $begingroup$
    Well, there is $zeta(3) = -Psi''(1)/2$, but I don't know if you'd count that as "better-known".
    $endgroup$
    – Robert Israel
    Nov 5 '18 at 2:26










  • $begingroup$
    "The exact value of ζ(3) is ζ(3)" isn't exactly an enlightening answer." Why not? Why would not you say "the exact value of $pi$ isn't quite known"?
    $endgroup$
    – user
    Nov 5 '18 at 22:07





















2












$begingroup$

Well, $sum_{ngeq 1}frac{1}{n^3}$ is a number, precisely the value of the Riemann $zeta$ function at $s=3$. Since $xleq text{arctanh}(x)$ is a pretty tight approximation for $xto 0^+$,



$$zeta(3) = 1+sum_{ngeq 2}frac{1}{n^3} leq 1+frac{1}{2}sum_{ngeq 2}logleft(frac{n^3+1}{n^3-1}right)$$
but $prod_{ngeq 2}frac{n^3+1}{n^3-1}=prod_{ngeq 2}frac{n+1}{n-1}cdotfrac{n^2-n+1}{n^2+n+1}$ is a telescopic product convergent to $frac{3}{2}$, hence
$$ zeta(3) approx 1+frac{log 3-log 2}{2}.$$
By creative telescoping we also have the acceleration formula
$$ zeta(3)=sum_{ngeq 1}frac{1}{n^3}=frac{5}{2}sum_{ngeq 1}frac{(-1)^{n+1}}{n^3binom{2n}{n}}$$
(see my notes for a proof) which allowed Apery to prove that $zeta(3)notinmathbb{Q}$.

The irrationality of $zeta(5),zeta(7),zeta(9),ldots$ still is an open problem, like the conjecture $zeta(3)inpi^3mathbb{Q}$, which looks numerically very unlikely. On the other hand $sum_{ngeq 0}frac{(-1)^n}{(2n+1)^3}inpi^3mathbb{Q}$, as shown here.






share|cite|improve this answer









$endgroup$













    Your Answer





    StackExchange.ifUsing("editor", function () {
    return StackExchange.using("mathjaxEditing", function () {
    StackExchange.MarkdownEditor.creationCallbacks.add(function (editor, postfix) {
    StackExchange.mathjaxEditing.prepareWmdForMathJax(editor, postfix, [["$", "$"], ["\\(","\\)"]]);
    });
    });
    }, "mathjax-editing");

    StackExchange.ready(function() {
    var channelOptions = {
    tags: "".split(" "),
    id: "69"
    };
    initTagRenderer("".split(" "), "".split(" "), channelOptions);

    StackExchange.using("externalEditor", function() {
    // Have to fire editor after snippets, if snippets enabled
    if (StackExchange.settings.snippets.snippetsEnabled) {
    StackExchange.using("snippets", function() {
    createEditor();
    });
    }
    else {
    createEditor();
    }
    });

    function createEditor() {
    StackExchange.prepareEditor({
    heartbeatType: 'answer',
    autoActivateHeartbeat: false,
    convertImagesToLinks: true,
    noModals: true,
    showLowRepImageUploadWarning: true,
    reputationToPostImages: 10,
    bindNavPrevention: true,
    postfix: "",
    imageUploader: {
    brandingHtml: "Powered by u003ca class="icon-imgur-white" href="https://imgur.com/"u003eu003c/au003e",
    contentPolicyHtml: "User contributions licensed under u003ca href="https://creativecommons.org/licenses/by-sa/3.0/"u003ecc by-sa 3.0 with attribution requiredu003c/au003e u003ca href="https://stackoverflow.com/legal/content-policy"u003e(content policy)u003c/au003e",
    allowUrls: true
    },
    noCode: true, onDemand: true,
    discardSelector: ".discard-answer"
    ,immediatelyShowMarkdownHelp:true
    });


    }
    });














    draft saved

    draft discarded


















    StackExchange.ready(
    function () {
    StackExchange.openid.initPostLogin('.new-post-login', 'https%3a%2f%2fmath.stackexchange.com%2fquestions%2f2984928%2fwhat-does-1-frac18-frac127-frac164-frac1125-frac1n3%23new-answer', 'question_page');
    }
    );

    Post as a guest















    Required, but never shown

























    2 Answers
    2






    active

    oldest

    votes








    2 Answers
    2






    active

    oldest

    votes









    active

    oldest

    votes






    active

    oldest

    votes









    2












    $begingroup$

    $zeta(3)$ is a constant known as Apery's constant. Its value is approximately $1.2021ldots$ but as far as I know an exact value isn't known.



    Some further reading:



    https://en.wikipedia.org/wiki/Ap%C3%A9ry%27s_constant
    http://mathworld.wolfram.com/AperysConstant.html






    share|cite|improve this answer











    $endgroup$













    • $begingroup$
      I don't know what it is you think "isn't quite known". The "exact value" is $zeta(3)$. Apéry proved it is irrational.
      $endgroup$
      – Robert Israel
      Nov 4 '18 at 22:49










    • $begingroup$
      "The exact value of $zeta(3)$ is $zeta(3)$" isn't exactly an enlightening answer. I mean something in terms of - at least more known - constants, i.e. how we know $zeta(2) = pi^2/6$. A "closed form" value, perhaps? Whatever the term is.
      $endgroup$
      – Eevee Trainer
      Nov 4 '18 at 22:51










    • $begingroup$
      It is not likely that $zeta(3)$ has a closed form in terms of "better-known" constants, though of course this has not been proven. We don't even have a proof that it is transcendental.
      $endgroup$
      – Robert Israel
      Nov 4 '18 at 23:00










    • $begingroup$
      Well, there is $zeta(3) = -Psi''(1)/2$, but I don't know if you'd count that as "better-known".
      $endgroup$
      – Robert Israel
      Nov 5 '18 at 2:26










    • $begingroup$
      "The exact value of ζ(3) is ζ(3)" isn't exactly an enlightening answer." Why not? Why would not you say "the exact value of $pi$ isn't quite known"?
      $endgroup$
      – user
      Nov 5 '18 at 22:07


















    2












    $begingroup$

    $zeta(3)$ is a constant known as Apery's constant. Its value is approximately $1.2021ldots$ but as far as I know an exact value isn't known.



    Some further reading:



    https://en.wikipedia.org/wiki/Ap%C3%A9ry%27s_constant
    http://mathworld.wolfram.com/AperysConstant.html






    share|cite|improve this answer











    $endgroup$













    • $begingroup$
      I don't know what it is you think "isn't quite known". The "exact value" is $zeta(3)$. Apéry proved it is irrational.
      $endgroup$
      – Robert Israel
      Nov 4 '18 at 22:49










    • $begingroup$
      "The exact value of $zeta(3)$ is $zeta(3)$" isn't exactly an enlightening answer. I mean something in terms of - at least more known - constants, i.e. how we know $zeta(2) = pi^2/6$. A "closed form" value, perhaps? Whatever the term is.
      $endgroup$
      – Eevee Trainer
      Nov 4 '18 at 22:51










    • $begingroup$
      It is not likely that $zeta(3)$ has a closed form in terms of "better-known" constants, though of course this has not been proven. We don't even have a proof that it is transcendental.
      $endgroup$
      – Robert Israel
      Nov 4 '18 at 23:00










    • $begingroup$
      Well, there is $zeta(3) = -Psi''(1)/2$, but I don't know if you'd count that as "better-known".
      $endgroup$
      – Robert Israel
      Nov 5 '18 at 2:26










    • $begingroup$
      "The exact value of ζ(3) is ζ(3)" isn't exactly an enlightening answer." Why not? Why would not you say "the exact value of $pi$ isn't quite known"?
      $endgroup$
      – user
      Nov 5 '18 at 22:07
















    2












    2








    2





    $begingroup$

    $zeta(3)$ is a constant known as Apery's constant. Its value is approximately $1.2021ldots$ but as far as I know an exact value isn't known.



    Some further reading:



    https://en.wikipedia.org/wiki/Ap%C3%A9ry%27s_constant
    http://mathworld.wolfram.com/AperysConstant.html






    share|cite|improve this answer











    $endgroup$



    $zeta(3)$ is a constant known as Apery's constant. Its value is approximately $1.2021ldots$ but as far as I know an exact value isn't known.



    Some further reading:



    https://en.wikipedia.org/wiki/Ap%C3%A9ry%27s_constant
    http://mathworld.wolfram.com/AperysConstant.html







    share|cite|improve this answer














    share|cite|improve this answer



    share|cite|improve this answer








    edited Dec 13 '18 at 9:02

























    answered Nov 4 '18 at 22:29









    Eevee TrainerEevee Trainer

    6,80311237




    6,80311237












    • $begingroup$
      I don't know what it is you think "isn't quite known". The "exact value" is $zeta(3)$. Apéry proved it is irrational.
      $endgroup$
      – Robert Israel
      Nov 4 '18 at 22:49










    • $begingroup$
      "The exact value of $zeta(3)$ is $zeta(3)$" isn't exactly an enlightening answer. I mean something in terms of - at least more known - constants, i.e. how we know $zeta(2) = pi^2/6$. A "closed form" value, perhaps? Whatever the term is.
      $endgroup$
      – Eevee Trainer
      Nov 4 '18 at 22:51










    • $begingroup$
      It is not likely that $zeta(3)$ has a closed form in terms of "better-known" constants, though of course this has not been proven. We don't even have a proof that it is transcendental.
      $endgroup$
      – Robert Israel
      Nov 4 '18 at 23:00










    • $begingroup$
      Well, there is $zeta(3) = -Psi''(1)/2$, but I don't know if you'd count that as "better-known".
      $endgroup$
      – Robert Israel
      Nov 5 '18 at 2:26










    • $begingroup$
      "The exact value of ζ(3) is ζ(3)" isn't exactly an enlightening answer." Why not? Why would not you say "the exact value of $pi$ isn't quite known"?
      $endgroup$
      – user
      Nov 5 '18 at 22:07




















    • $begingroup$
      I don't know what it is you think "isn't quite known". The "exact value" is $zeta(3)$. Apéry proved it is irrational.
      $endgroup$
      – Robert Israel
      Nov 4 '18 at 22:49










    • $begingroup$
      "The exact value of $zeta(3)$ is $zeta(3)$" isn't exactly an enlightening answer. I mean something in terms of - at least more known - constants, i.e. how we know $zeta(2) = pi^2/6$. A "closed form" value, perhaps? Whatever the term is.
      $endgroup$
      – Eevee Trainer
      Nov 4 '18 at 22:51










    • $begingroup$
      It is not likely that $zeta(3)$ has a closed form in terms of "better-known" constants, though of course this has not been proven. We don't even have a proof that it is transcendental.
      $endgroup$
      – Robert Israel
      Nov 4 '18 at 23:00










    • $begingroup$
      Well, there is $zeta(3) = -Psi''(1)/2$, but I don't know if you'd count that as "better-known".
      $endgroup$
      – Robert Israel
      Nov 5 '18 at 2:26










    • $begingroup$
      "The exact value of ζ(3) is ζ(3)" isn't exactly an enlightening answer." Why not? Why would not you say "the exact value of $pi$ isn't quite known"?
      $endgroup$
      – user
      Nov 5 '18 at 22:07


















    $begingroup$
    I don't know what it is you think "isn't quite known". The "exact value" is $zeta(3)$. Apéry proved it is irrational.
    $endgroup$
    – Robert Israel
    Nov 4 '18 at 22:49




    $begingroup$
    I don't know what it is you think "isn't quite known". The "exact value" is $zeta(3)$. Apéry proved it is irrational.
    $endgroup$
    – Robert Israel
    Nov 4 '18 at 22:49












    $begingroup$
    "The exact value of $zeta(3)$ is $zeta(3)$" isn't exactly an enlightening answer. I mean something in terms of - at least more known - constants, i.e. how we know $zeta(2) = pi^2/6$. A "closed form" value, perhaps? Whatever the term is.
    $endgroup$
    – Eevee Trainer
    Nov 4 '18 at 22:51




    $begingroup$
    "The exact value of $zeta(3)$ is $zeta(3)$" isn't exactly an enlightening answer. I mean something in terms of - at least more known - constants, i.e. how we know $zeta(2) = pi^2/6$. A "closed form" value, perhaps? Whatever the term is.
    $endgroup$
    – Eevee Trainer
    Nov 4 '18 at 22:51












    $begingroup$
    It is not likely that $zeta(3)$ has a closed form in terms of "better-known" constants, though of course this has not been proven. We don't even have a proof that it is transcendental.
    $endgroup$
    – Robert Israel
    Nov 4 '18 at 23:00




    $begingroup$
    It is not likely that $zeta(3)$ has a closed form in terms of "better-known" constants, though of course this has not been proven. We don't even have a proof that it is transcendental.
    $endgroup$
    – Robert Israel
    Nov 4 '18 at 23:00












    $begingroup$
    Well, there is $zeta(3) = -Psi''(1)/2$, but I don't know if you'd count that as "better-known".
    $endgroup$
    – Robert Israel
    Nov 5 '18 at 2:26




    $begingroup$
    Well, there is $zeta(3) = -Psi''(1)/2$, but I don't know if you'd count that as "better-known".
    $endgroup$
    – Robert Israel
    Nov 5 '18 at 2:26












    $begingroup$
    "The exact value of ζ(3) is ζ(3)" isn't exactly an enlightening answer." Why not? Why would not you say "the exact value of $pi$ isn't quite known"?
    $endgroup$
    – user
    Nov 5 '18 at 22:07






    $begingroup$
    "The exact value of ζ(3) is ζ(3)" isn't exactly an enlightening answer." Why not? Why would not you say "the exact value of $pi$ isn't quite known"?
    $endgroup$
    – user
    Nov 5 '18 at 22:07













    2












    $begingroup$

    Well, $sum_{ngeq 1}frac{1}{n^3}$ is a number, precisely the value of the Riemann $zeta$ function at $s=3$. Since $xleq text{arctanh}(x)$ is a pretty tight approximation for $xto 0^+$,



    $$zeta(3) = 1+sum_{ngeq 2}frac{1}{n^3} leq 1+frac{1}{2}sum_{ngeq 2}logleft(frac{n^3+1}{n^3-1}right)$$
    but $prod_{ngeq 2}frac{n^3+1}{n^3-1}=prod_{ngeq 2}frac{n+1}{n-1}cdotfrac{n^2-n+1}{n^2+n+1}$ is a telescopic product convergent to $frac{3}{2}$, hence
    $$ zeta(3) approx 1+frac{log 3-log 2}{2}.$$
    By creative telescoping we also have the acceleration formula
    $$ zeta(3)=sum_{ngeq 1}frac{1}{n^3}=frac{5}{2}sum_{ngeq 1}frac{(-1)^{n+1}}{n^3binom{2n}{n}}$$
    (see my notes for a proof) which allowed Apery to prove that $zeta(3)notinmathbb{Q}$.

    The irrationality of $zeta(5),zeta(7),zeta(9),ldots$ still is an open problem, like the conjecture $zeta(3)inpi^3mathbb{Q}$, which looks numerically very unlikely. On the other hand $sum_{ngeq 0}frac{(-1)^n}{(2n+1)^3}inpi^3mathbb{Q}$, as shown here.






    share|cite|improve this answer









    $endgroup$


















      2












      $begingroup$

      Well, $sum_{ngeq 1}frac{1}{n^3}$ is a number, precisely the value of the Riemann $zeta$ function at $s=3$. Since $xleq text{arctanh}(x)$ is a pretty tight approximation for $xto 0^+$,



      $$zeta(3) = 1+sum_{ngeq 2}frac{1}{n^3} leq 1+frac{1}{2}sum_{ngeq 2}logleft(frac{n^3+1}{n^3-1}right)$$
      but $prod_{ngeq 2}frac{n^3+1}{n^3-1}=prod_{ngeq 2}frac{n+1}{n-1}cdotfrac{n^2-n+1}{n^2+n+1}$ is a telescopic product convergent to $frac{3}{2}$, hence
      $$ zeta(3) approx 1+frac{log 3-log 2}{2}.$$
      By creative telescoping we also have the acceleration formula
      $$ zeta(3)=sum_{ngeq 1}frac{1}{n^3}=frac{5}{2}sum_{ngeq 1}frac{(-1)^{n+1}}{n^3binom{2n}{n}}$$
      (see my notes for a proof) which allowed Apery to prove that $zeta(3)notinmathbb{Q}$.

      The irrationality of $zeta(5),zeta(7),zeta(9),ldots$ still is an open problem, like the conjecture $zeta(3)inpi^3mathbb{Q}$, which looks numerically very unlikely. On the other hand $sum_{ngeq 0}frac{(-1)^n}{(2n+1)^3}inpi^3mathbb{Q}$, as shown here.






      share|cite|improve this answer









      $endgroup$
















        2












        2








        2





        $begingroup$

        Well, $sum_{ngeq 1}frac{1}{n^3}$ is a number, precisely the value of the Riemann $zeta$ function at $s=3$. Since $xleq text{arctanh}(x)$ is a pretty tight approximation for $xto 0^+$,



        $$zeta(3) = 1+sum_{ngeq 2}frac{1}{n^3} leq 1+frac{1}{2}sum_{ngeq 2}logleft(frac{n^3+1}{n^3-1}right)$$
        but $prod_{ngeq 2}frac{n^3+1}{n^3-1}=prod_{ngeq 2}frac{n+1}{n-1}cdotfrac{n^2-n+1}{n^2+n+1}$ is a telescopic product convergent to $frac{3}{2}$, hence
        $$ zeta(3) approx 1+frac{log 3-log 2}{2}.$$
        By creative telescoping we also have the acceleration formula
        $$ zeta(3)=sum_{ngeq 1}frac{1}{n^3}=frac{5}{2}sum_{ngeq 1}frac{(-1)^{n+1}}{n^3binom{2n}{n}}$$
        (see my notes for a proof) which allowed Apery to prove that $zeta(3)notinmathbb{Q}$.

        The irrationality of $zeta(5),zeta(7),zeta(9),ldots$ still is an open problem, like the conjecture $zeta(3)inpi^3mathbb{Q}$, which looks numerically very unlikely. On the other hand $sum_{ngeq 0}frac{(-1)^n}{(2n+1)^3}inpi^3mathbb{Q}$, as shown here.






        share|cite|improve this answer









        $endgroup$



        Well, $sum_{ngeq 1}frac{1}{n^3}$ is a number, precisely the value of the Riemann $zeta$ function at $s=3$. Since $xleq text{arctanh}(x)$ is a pretty tight approximation for $xto 0^+$,



        $$zeta(3) = 1+sum_{ngeq 2}frac{1}{n^3} leq 1+frac{1}{2}sum_{ngeq 2}logleft(frac{n^3+1}{n^3-1}right)$$
        but $prod_{ngeq 2}frac{n^3+1}{n^3-1}=prod_{ngeq 2}frac{n+1}{n-1}cdotfrac{n^2-n+1}{n^2+n+1}$ is a telescopic product convergent to $frac{3}{2}$, hence
        $$ zeta(3) approx 1+frac{log 3-log 2}{2}.$$
        By creative telescoping we also have the acceleration formula
        $$ zeta(3)=sum_{ngeq 1}frac{1}{n^3}=frac{5}{2}sum_{ngeq 1}frac{(-1)^{n+1}}{n^3binom{2n}{n}}$$
        (see my notes for a proof) which allowed Apery to prove that $zeta(3)notinmathbb{Q}$.

        The irrationality of $zeta(5),zeta(7),zeta(9),ldots$ still is an open problem, like the conjecture $zeta(3)inpi^3mathbb{Q}$, which looks numerically very unlikely. On the other hand $sum_{ngeq 0}frac{(-1)^n}{(2n+1)^3}inpi^3mathbb{Q}$, as shown here.







        share|cite|improve this answer












        share|cite|improve this answer



        share|cite|improve this answer










        answered Nov 4 '18 at 23:31









        Jack D'AurizioJack D'Aurizio

        290k33282664




        290k33282664






























            draft saved

            draft discarded




















































            Thanks for contributing an answer to Mathematics Stack Exchange!


            • Please be sure to answer the question. Provide details and share your research!

            But avoid



            • Asking for help, clarification, or responding to other answers.

            • Making statements based on opinion; back them up with references or personal experience.


            Use MathJax to format equations. MathJax reference.


            To learn more, see our tips on writing great answers.




            draft saved


            draft discarded














            StackExchange.ready(
            function () {
            StackExchange.openid.initPostLogin('.new-post-login', 'https%3a%2f%2fmath.stackexchange.com%2fquestions%2f2984928%2fwhat-does-1-frac18-frac127-frac164-frac1125-frac1n3%23new-answer', 'question_page');
            }
            );

            Post as a guest















            Required, but never shown





















































            Required, but never shown














            Required, but never shown












            Required, but never shown







            Required, but never shown

































            Required, but never shown














            Required, but never shown












            Required, but never shown







            Required, but never shown







            Popular posts from this blog

            Le Mesnil-Réaume

            Ida-Boy-Ed-Garten

            web3.py web3.isConnected() returns false always